Đến nội dung

Simpson Joe Donald nội dung

Có 290 mục bởi Simpson Joe Donald (Tìm giới hạn từ 10-05-2020)



Sắp theo                Sắp xếp  

#492381 Trận 7 - PT, HPT đại số

Đã gửi bởi Simpson Joe Donald on 12-04-2014 - 11:35 trong Thi giải toán Marathon cấp THCS 2014

Mở rộng:  Giải phương trình: $a. A(x)+b.B(x)=c.\sqrt{A(x).B(x)}$ với $a,b,c\in \mathbb{R}; \ A(x), \ B(x)$ là các đa thức.

Giải: Đặt $\begin{cases}\sqrt{A(x)}=m\ge 0\\\sqrt{B(x)}=n\ge 0\end{cases}$. Phương trình có dạng:

$$am^2+bn^2=cmn \ \ (*)$$

Nhận thấy $n=0$ không là nghiệm của phương trình , chia cả 2 vế phương trình $(*)$ cho $n^2\neq 0$ ta được:

$$a.\dfrac{m^2}{n^2}-c.\dfrac{m}{n}+b=0\iff at^2-ct+b=0\left(t=\dfrac{m}{n}\right) \implies t=\dfrac{c\pm \sqrt{\Delta}}{2a}$$

Tìm được $t$ thay vào tìm tỉ số $m,n$ rồi tìm ra x. 

 




#492384 Trận 7 - PT, HPT đại số

Đã gửi bởi Simpson Joe Donald on 12-04-2014 - 11:39 trong Thi giải toán Marathon cấp THCS 2014

Giải phương trình: $2x^{2}+5x-1=7\sqrt{x^{3}-1}$

Đề thi của l4lzTeoz

Cách 2:    ĐK: $x\ge 1$, bình phương 2 vế :

$$(2x^{2}+5x-1)^2=(7\sqrt{x^{3}-1})^2 \\ \iff 4x^4-29x^3+21x^2-10x+50=0 \\ \iff (x^2-8x+10)(4x^2+3x+5)=0 \\  \implies x=4\pm \sqrt{6}$$




#492373 Trận 7 - PT, HPT đại số

Đã gửi bởi Simpson Joe Donald on 12-04-2014 - 11:13 trong Thi giải toán Marathon cấp THCS 2014

Giải phương trình: $2x^{2}+5x-1=7\sqrt{x^{3}-1}$

Đề thi của l4lzTeoz

ĐK: $x\geq 1$
Phương trình đã cho tương đương với: $$2(x^{2}+x+1)+3(x-1)=7\sqrt{(x-1)(x^{2}+x+1)}$$
Đặt $u=\sqrt{x^{2}+x+1},v=\sqrt{x-1}; \ u,v\ge 0 $ suy ra:

$$2u^{2}+3v^{2}=7uv \iff  (u-3v)(2u-v)=0 \\ \iff \begin{bmatrix} u=3v \\ 2u=v \end{bmatrix}  \iff \begin{bmatrix} \sqrt{x^2+x+1}=3\sqrt{x-1} \\ 2\sqrt{x^2+x+1}=\sqrt{x-1} \end{bmatrix} \\ \iff \begin{bmatrix} x^2-8x+10=0 \\ 4x^2+3x+5=0 (VN)\end{bmatrix} \iff \begin{bmatrix} x=4- \sqrt{6} \\ x=4+ \sqrt{6} \end{bmatrix}$$

Vậy Tập nghiệm của phương trình là: $S=\{ 4 \pm \sqrt{6} \}$




#483549 Trận 3 - Hình học

Đã gửi bởi Simpson Joe Donald on 16-02-2014 - 21:04 trong Thi giải toán Marathon cấp THCS 2014

Đầu tiên ta có bổ đề (*) : ( Định lý Ceva)

Cho một tam giác ABC, các điểm D, E, và F lần lượt nằm trên các đường thẳng BC, CA, và AB. ĐỊnh lý phát biểu rằng các đường thẳng AD, BE và CF là những đường thẳng đồng quy khi và chỉ khi: $\frac{AF}{FB}.\frac{BD}{DC}.\frac{CE}{EA}=1$      

Chứng minh: 

File gửi kèm  MSS.1.bmp   115.52K   1 Số lần tải    

Giả sử $AD; \ BE ; \ CF$ đồng quy tại 1 điểm $O$ nào đó ( nằm trong hoặc ngoài tam giác).

Ta có: $\Delta BOD$ và $\Delta COD$ có chung chiều cao $\implies \frac{|\Delta BOD|}{|\Delta COD|}=\frac{BD}{DC}$

Tương tự $\frac{|\Delta BAD|}{|\Delta CAD|}=\frac{BD}{DC}$

Ta suy ra: $$\frac{BD}{DC}=\frac{|\Delta BAD|-|\Delta BOD|}{|\Delta CAD|-|\Delta COD|}=\frac{|\Delta ABO|}{|\Delta CAO|}$$

Tương tự: $\frac{CE}{EA}=\frac{|\Delta BCO|}{|\Delta ABO|}$ và $\frac{AF}{FB}=\frac{|\Delta CAO|}{|\Delta BCO|}$

Nhân 3 đẳng thức trên ta được: $\frac{AF}{FB}.\frac{BD}{DC}.\frac{CE}{EA}=1$       

 

Ngược lại , giả sử Ngược lại, giả sử rằng ta đã có những điểm $D, E$ và $F$ thỏa mãn đẳng thức. Gọi giao điểm của $AD$ và $BE$ là $O$, và gọi giao điểm của $CO$ và $AB$ là $F'$. Theo chứng minh trên $\frac{AF'}{F'B}.\frac{BD}{DC}.\frac{CE}{EA}=1$

Kết hợp với đẳng thức trên, ta nhận được: $\frac{AF'}{F'B}=\frac{AF}{FB}$

Thêm 1 vào mỗi vế và chú ý rằng $AF''+F''B=AF+FB=AB$, ta có $\frac{AB}{F'B}=\frac{AB}{FB}$

Do đó $F''B=FB$, vậy $F$ và $F''$ trùng nhau. Vì vậy $AD,BE$ và $CF=CF''$ đồng qui tại $O$, và định lí đã được chứng minh đúng theo cả hai chiều.

 

Trở về bài toán: 

 

File gửi kèm  MSS.3.bmp   680.89K   4 Số lần tải

 

$AM$ cắt $BC$ tại $K$, $BN$ cắt $AC$ tại $J$, $CP$ cắt $AB$ tại H.

 

Kẻ $EI//FL//BC$ ( $F,I\in AK$) thì ta có : $\frac{AF}{AB}=\frac{FL}{BK} \ ; \ \frac{AE}{AC}=\frac{EI}{CK} \ ; \ \frac{EI}{FL}=\frac{EM}{FM}$

 

Nhân 3 đẳng thức trên ta có: $\frac{AB}{AF}.\frac{AE}{AC}.\frac{FM}{EM}=\frac{BK}{FL}.\frac{EI}{CK}.\frac{FL}{EI}\implies \frac{BK}{CK}= \frac{FM}{EM} .\frac{AB}{AF}.\frac{AE}{AC} \ \ (1)$

 

Tượng tự ta cũng có: $\frac{CJ}{AJ}=\frac{DN}{FN}.\frac{BC}{BD}.\frac{BF}{AB} \ \ (2)$ và $\frac{AH}{BH}=\frac{EP}{DP}.\frac{AC}{EC}.\frac{CD}{BC} \ \ \ (3)$

 

Nhân các vế của $(1);(2);(3)$ ta được:

 

$\frac{BK}{CK}.\frac{CJ}{AJ}.\frac{AH}{BH}=\frac{AB}{AF}.\frac{AE}{AC}.\frac{FM}{EM}.\frac{DN}{FN}.\frac{BC}{BD}.\frac{BF}{AB}.\frac{EP}{DP}.\frac{AC}{EC}.\frac{CD}{BC} \\ \implies \frac{BK}{CK}.\frac{CJ}{AJ}.\frac{AH}{BH}=\frac{FM}{EM}.\frac{EP}{DP}.\frac{DN}{FN}.\frac{DC}{BD}.\frac{BF}{AF}.\frac{AE}{CE} \ \ (\star)$

 

Áp dụng bổ đề (*) cho $\Delta ABD$ có $AD, CE, CF$ đồng quy và $\Delta DEF$ có $DM,EN,FP$ đồng quy ta có:

$\frac{FM}{EM}.\frac{EP}{DP}.\frac{DN}{FN}=1$ và $\frac{DC}{BD}.\frac{BF}{AF}.\frac{AE}{CE}=1$

Thay vào $(\star)$ : $\implies \frac{BK}{CK}.\frac{CJ}{AJ}.\frac{AH}{BH}=1$

Theo bổ đề (*) thì ta có điều phải chứng minh. $\blacksquare$

 

$d=10$

$S= 39$




#476867 $\begin{cases}x^3-3x^2+2x-5=y \\ y^3+3y^2+2y-4=z\end...

Đã gửi bởi Simpson Joe Donald on 12-01-2014 - 11:37 trong Phương trình, hệ phương trình và bất phương trình

vẫn còn phần a) bác nào giải hộ với

giải hệ $\left\{\begin{matrix} 36x^{2}y-60x^{2}+25z=0\\ 36y^{2}z-60y^{2}+25x=0\\ 36z^{2}x-60z^{2}+25y=0 \end{matrix}\right.$

$y=\frac{60x^2}{36x^2+25}$ nên $y\geq 0$. Tương tự $x,z\geq0$.

$TH_1:$ Dễ thấy $x=y=z=0$ là nghiệm.

$TH_2:$ Xét $x,y,z>0$.

Hệ trên suy ra:$$xyz=\frac{60^3x^2y^2z^2}{(36x^2+25)(36y^2+25)(36z^2+25)}$$
$$\Leftrightarrow 1=\frac{60^3xyz}{(36x^2+25)(36y^2+25)(36z^2+25)}\leq \frac{60^3xyz}{(2.6.5.x)(2.6.5.y)(2.6.5.z)}=1$$
Dấu bằng xảy ra khi $a=b=c=\frac{5}{6}$

Vậy hệ có nghiệm $(0;0;0)$ và $\left ( \frac{5}{6};\frac{5}{6};\frac{5}{6} \right )$




#440176 Tôpic nhận đề Phương trình nghiệm nguyên, đồng dư, chia hết.

Đã gửi bởi Simpson Joe Donald on 03-08-2013 - 17:15 trong Bài thi đang diễn ra

Họ tện: Dương Bá Linh

Lớp: 9

Trường thcs thọ thế.

Đề bài:

Tìm số bị chia, số chia và thương trong phép chia sau:
                   $$\overline{abcd} : \overline{dcba}= q$$
Biết 3 số đều là số chính phương.
Đáp án:
Vì 3 số đều là số chính phương nên ta có $\overline{abcd}=m^2 \ ; \ \overline{dcba}=n^2 \ ; \ q \in \{1;4;9\}$
$TH_1:$ Với $  q=1 \Rightarrow \overline{abcd}=\overline{dcba} \ \  (\text{loại})$
$TH_2:$ Với $ q=4 \Rightarrow \overline{abcd}=4\overline{dcba}$
$\Rightarrow m^2=4n^2 \Rightarrow m=2n $
 $\overline{abcd} \leq 9999 \Rightarrow \overline{dcba} \leq 2499 \Rightarrow d\in \{1;2\}$
$+, \ d=1 \Rightarrow$ m tận cùng là 1 (loại) vì m phải chẵn
$+,  \ d=2 \Rightarrow$ loại vì $\overline{abcd}$ chính phương
 
$TH_3:$ Với  $q=9 \Rightarrow \overline{abcd}=9\overline{dcba}$
 $\Rightarrow m=3n$
$\overline {abcd} \leq 9999 \Rightarrow \overline{dcba} \leq 1111$
$\Rightarrow d=1 \Rightarrow m$ tận cùng là 1 hoặc 9
$\Rightarrow n$ tận cùng là 7 hoặc 3
$\Rightarrow a=9$
Ta có:
$\overline{9bc1}=9.\overline{1cb9}$
$ \Leftrightarrow 9000+\overline{bc}.10+1=9(1000+\overline{cb}.10+9)$
 $\Leftrightarrow 81c=b-8$
 $\Rightarrow c=0,b=8$
Vậy số cần tìm là :  $\boxed{\overline{abcd}=9801 \ ; \ \overline{dcba}=1089 \ ; \ q=9}$

 

 

 

 




#474065 Giải phương trình: $(x+3)\sqrt{-x^{2}-8x+48}=x-...

Đã gửi bởi Simpson Joe Donald on 30-12-2013 - 23:17 trong Phương trình - hệ phương trình - bất phương trình

Có chút nhầm lẫn rồi bạn ơi

Nhìn nhầm đề nên giải sai đã fix




#474062 Giải phương trình: $(x+3)\sqrt{-x^{2}-8x+48}=x-...

Đã gửi bởi Simpson Joe Donald on 30-12-2013 - 23:10 trong Phương trình - hệ phương trình - bất phương trình

Bài 1: Giải phương trình:

$(x+3)\sqrt{-x^{2}-8x+48}=x-24$

 

Đặt $t=\sqrt{-x^2-8x+48}$. Điều kiện: $t\geq 0$
Biến đổi phương trình tương đương
$$t^2+2(x+3)t+x^2+6x=0$$
Từ đó, $t=-x$ hoặc $t=-x-6$

 




#441433 Tìm các số nguyên không âm $x,y$ thoả mãn đẳng thức $x^2=y^2+...

Đã gửi bởi Simpson Joe Donald on 09-08-2013 - 10:29 trong Số học



 

Bài 7: Tìm số $n$ nguyên dương thoả mãn $\sqrt{(3+2\sqrt{2})^{n}}+\sqrt{(3-2\sqrt{2})^{n}}=6$

 

Nhận xét: $(3+2\sqrt{2})(3-2\sqrt{2})=1$

Đặt: $\sqrt{(3-2\sqrt{2})^x}=a>0$

PT trở thành:

$a+\dfrac{1}{a}=6\iff a^2-6a+1=0 \iff \left[ \begin{array}{l} a=3-2\sqrt{2} \\ a=3+2\sqrt{2} \end{array}\right.$




#492659 Trận 7 - PT, BPT

Đã gửi bởi Simpson Joe Donald on 13-04-2014 - 15:04 trong Thi giải toán Marathon cấp THPT 2014

Giải phương trình:

$$\sqrt{x^2+x-6}+3\sqrt{x-1}=\sqrt{3x^2-6x+19}$$

Đề của 

vuminhhoang

Em không phải toán thủ thi đấu nhưng làm góp vui!
ĐK ; $x\ge 2$
$$pt\iff  6\sqrt{(x-1)(x-2)(x+3)}=2x^2-16x+34 \\ \iff  6\sqrt{(x^2+2x-3)(x-2)}=2(x^2+2x-3)-20(x-2)$$
Đặt $\sqrt{x^2+2x-3}=a; \ \sqrt{x-2}=b; \  \ a,b\ge 0$, pt có dạng:
$$3ab=a^2-10b\iff (a-5b)(a+2b)=0$$
Đến đây dễ dàng làm tiếp :P



#478423 Chứng minh rằng : $\frac{3}{xy+yz+xz}+\fr...

Đã gửi bởi Simpson Joe Donald on 21-01-2014 - 22:35 trong Đại số

Cho $x,y,z\geq 0$ thỏa mãn $x+y+z=1$. Chứng minh rằng : 

$\frac{3}{xy+yz+xz}+\frac{2}{x^2+y^2+z^2}\geq14$

Đặt $t=xy+yz+zx$
$\implies x^2+y^2+z^2=(x+y+z)^2-2(xy+yz+zx)=1-2t$
Ta có:
$t=xy+yz+zx=\dfrac{2(xy+yz+zx)+(xy+yz+zx)}{3} \le \dfrac{(x+y+z)^2}{3}=\dfrac{1}{3}$
Giờ ta chỉ phải chứng minh $\dfrac{3}{t}+\dfrac{2}{1-2t}\ge 14$ với $0\le t\le \dfrac{1}{3}$.
Đơn giản là ta chỉ việc biến đổi tương đương, ta có:
$BDT\iff 3(1-2t)+2t\ge 14t(1-2t)\iff 3-18t+28t^2\ge 0 \iff 3(1-3t)^2+t^2\ge 0$
Phép chứng minh hoàn tất. $\blacksquare$



#478426 Chứng minh rằng : $\frac{3}{xy+yz+xz}+\fr...

Đã gửi bởi Simpson Joe Donald on 21-01-2014 - 22:38 trong Đại số

làm như 2 bạn ko có dấu =

Thì bài này thực ra không có dấu bằng mà!!!




#431377 Đề thi tuyển sinh trường THPT Chuyên Trần Hưng Đạo (Bình Thuận) năm 2013 - 2014

Đã gửi bởi Simpson Joe Donald on 28-06-2013 - 20:35 trong Tài liệu - Đề thi

 

 Bài 2 : (2 điểm)

Giải các phương trình và hệ phương trình sau:

1/ Chứng minh $\left\{\begin{matrix} x^{2}+y^{2}+x+y=18\\ x(x+1).y(y+1)=72 \end{matrix}\right.$

2/ $\sqrt{x^{2}-16}=3\sqrt{x-4}$

1, $hpt\Leftrightarrow \left\{\begin{matrix}x(x+1)+y(y+1)=18 & \\ x(x+1).y(y+1)=72 & \end{matrix}\right.$

$\left\{\begin{matrix}x(x+1)=a & \\ y(y+1)=b & \end{matrix}\right.\Rightarrow \left\{\begin{matrix}a+b=18 & \\ ab=72 & \end{matrix}\right.$

Tìm a,b rồi thay vào tìm x,y

2, $\left\{\begin{matrix}x-4\geq 0 & \\ x^2-16=3(x-4) & \end{matrix}\right.\Leftrightarrow \left\{\begin{matrix}x\geq 4 & \\ x^2-3x-4=0 & \end{matrix}\right.\Leftrightarrow \left\{\begin{matrix}x\ge 4 & \\ \begin{bmatrix}x=-1 & \\ x=4 & \end{bmatrix} & \end{matrix}\right. \Leftrightarrow x=4$




#431370 Đề thi tuyển sinh trường THPT Chuyên Trần Hưng Đạo (Bình Thuận) năm 2013 - 2014

Đã gửi bởi Simpson Joe Donald on 28-06-2013 - 20:19 trong Tài liệu - Đề thi

Bài 1   :  (2 điểm)

1/ Chứng minh $\sqrt{1+\frac{1}{(x-1)^{2}}+\frac{1}{x^{2}}} = 1 + \frac{1}{x-1} - \frac{1}{x}$ với x > 1

2/ Tính giá trị M = $\sqrt{1+\frac{1}{1^{2}}+\frac{1}{2^{2}}}+\sqrt{1+\frac{1}{2^{2}}+\frac{1}{3^{2}}}+...+\sqrt{1+\frac{1}{2012^{2}}+\frac{1}{2013^{2}}}$

 

Đặt $A=\sqrt{1+\frac{1}{(x-1)^2}+\frac{1}{x^2}}$

Ta có:$A^2=1+\frac{1}{(x-1)^2}+\frac{1}{x^2}=\frac{x^2(x-1)^2+x^2+(x-1)^2}{x^2(x-1)^2} \\ \ \ A^2=\frac{x^2(x^2-2x+1+1)+(x-1)^2}{x^2(x-1)^2} \\ A^2=\frac{x^4-2x(x-1)+(x-1)^2}{x^2(x-1)^2} \\ A^2=\left ( \frac{x^2-x+1}{x(x-1)} \right )^2$

Do $x>0 \Rightarrow A>0 \Rightarrow A=\frac{x^2-x+1}{x(x-1)}=1+\frac{1}{x-1}-\frac{1}{x}$




#436942 Cho a, b, c > 0 và a + b + c = 1. Tính Min A, B ?

Đã gửi bởi Simpson Joe Donald on 21-07-2013 - 20:02 trong Đại số

Lớp 8 à.Thế chưa học C-S thì không làm được đâu em.

Lớp 8 nâng cao là biết Cauchy rồi. Bạn ấy chỉ không hiểu $\sum$ thôi.




#449111 Topic nhận đề Phương trình, hệ phương trình đại số

Đã gửi bởi Simpson Joe Donald on 09-09-2013 - 21:07 trong Bài thi đang diễn ra

1. Họ và tên: Đường Bá Linh

2. Đang học lớp 9A trường THCS Thọ Thế, Huyện Triệu Sơn, Tỉnh Thanh Hóa

3. Đề: Giải phương trình: $$x=\left(\sqrt{x+1}+1\right)\left(\sqrt{x+10}-4\right)$$

4. Giải:   $DK: \ x\ge -1$ Ta có: $x=0$ không là nghiệm của phương trình nên $\sqrt{x+1}-1\neq 0$

$pt\iff x\left(\sqrt{x+1}-1\right)=\left(\sqrt{(x+1)^2}-1\right)\left(\sqrt{x+10}-4\right) \\ \iff x\left(\sqrt{x+1}-\sqrt{x+10}+3\right)=0 \\ \iff \sqrt{x+1}+3=\sqrt{x+10} \\ \iff x+1+9+6\sqrt{x+1}=x+10 \\ \iff 6\sqrt{x+1}=0 \iff x=-1(\text{thỏa mãn điều kiện})$ Vậy $x=1$ là nghiệm duy nhất của phương trinh.




#476836 giải phương trình nghiệm nguyên: $x^{2}+17y^{2}+34xy...

Đã gửi bởi Simpson Joe Donald on 12-01-2014 - 09:45 trong Số học

giải phương trình nghiệm nguyên:

$x^{2}+17y^{2}+34xy+51(x+y)=1740$

Nhận xét: $1740 \equiv 6 \pmod{17} \implies x^2 \equiv 6 \pmod{17}$
Lại  thấy một số chính phương chia $17$ chỉ dư $0,1,2,4,8,9,13,15,16$ 
Nên vô nghiệm
 

 




#437023 $ \sum \frac{1}{a+3b} \geq \sum \frac{1}{2a+2b}...

Đã gửi bởi Simpson Joe Donald on 22-07-2013 - 06:18 trong Bất đẳng thức và cực trị

$Cho   a\geq b\geq c> 0$.  Chứng minh

$\frac{1}{a+3b}+\frac{1}{b+3c}+\frac{1}{c+3a}\geq \frac{1}{2a+2b}+\frac{1}{2b+2c}+\frac{1}{2c+2a}$

Do $a\ge b\ge c>0$ nên ta đưa về bài toán chứng minh:

$\frac{1}{a+3b}+\frac{1}{b+3c}+\frac{1}{c+3a}\geq \frac{1}{2a+b+c}+\frac{1}{2b+c+a}+\frac{1}{2c+a+b}$

Theo $Cauchy-Schwarz$ ta có:

$\dfrac{2}{a+3b}+\dfrac{1}{b+3c}+\dfrac{4}{c+3a} \\ \ge \dfrac{(2+1+4)^2}{2(a+3b)+(b+3c)+4(c+3a)} \\ = \dfrac{7}{2a+b+c}$




#436941 $ \sum \frac{1}{a+3b} \geq \sum \frac{1}{2a+2b}...

Đã gửi bởi Simpson Joe Donald on 21-07-2013 - 20:00 trong Bất đẳng thức và cực trị

$Cho   a\geq b\geq c> 0$.  Chứng minh

$\frac{1}{a+3b}+\frac{1}{b+3c}+\frac{1}{c+3a}\geq \frac{1}{2a+2b}+\frac{1}{2b+2c}+\frac{1}{2c+2a}$

Đề là:

$\frac{1}{a+3b}+\frac{1}{b+3c}+\frac{1}{c+3a}\geq \frac{1}{2a+b+c}+\frac{1}{2b+c+a}+\frac{1}{2c+a+b}$

Thì đúng hơn.




#439697 Chứng minh $\frac{a}{bc+1}+\frac{b...

Đã gửi bởi Simpson Joe Donald on 01-08-2013 - 15:41 trong Bất đẳng thức và cực trị

Anh làm câu 2 nha:

đặt a+b+2c=x

     2a+b+c=y

     a+b+3c=z 

Nên 4a=4(y+z-2x)

       8c=8(z-x)

       b+3c=5x-y

Nên bđt cần cm tương đương: $\frac{4(y+z-2x)}{x}+\frac{2x-y}{y}-\frac{8(x-z)}{z}=\frac{4y}{x}+\frac{4z}{x}-8+\frac{8x}{z}-8+\frac{3x}{y}-1=(\frac{4y}{x}+\frac{3x}{y})+(\frac{8x}{z}+\frac{4z}{x})-17\geq 2\sqrt{12}+2\sqrt{32}-17$.

 

 

 

 

                                       :nav: Uchiha Itachi  :nav: 

Đã có tại đây: http://diendantoanho...geq-12sqrt2-17/




#430321 Cho các số dương a,b,c,d CMR: $\frac{a}{b+c} +...

Đã gửi bởi Simpson Joe Donald on 24-06-2013 - 20:35 trong Bất đẳng thức và cực trị

$$VT=\left (\frac{a}{b+c}+\frac{c}{a+d}  \right )+\left ( \frac{b}{d+c} +\frac{d}{a+b}\right )$$
 
$$=\frac{d(d+a)+c(b+c)}{(b+c)(d+a)}+\frac{b(a+b)+d(c+d)}{(a+b)(c+d)}$$
 
Áp dụng BĐT $$\frac{1}{xy}\geq \frac{4}{(x+y)^2}$$ Ta được:
 
$$VT\geq \frac{a^2+ad+bc+c^2}{(a+b+c+d)^2}+4.\frac{b^2+ad+cd+d^2}{(a+b+c+d)^2}$$
 
$$\geq 2.\frac{(a+b+c+d)^2+(a-c)^2+(b-d)^2}{(a+b+c+d)^2}\geq 2$$



#440114 $\frac{4}{a} + \sqrt[3]{4-b} =...

Đã gửi bởi Simpson Joe Donald on 03-08-2013 - 15:08 trong Phương trình, hệ phương trình và bất phương trình

$\fbox{1}. \ \dfrac{4}{a}+\sqrt[3]{4-b}=\sqrt[3]{4+4\sqrt{b}+b}+\sqrt[3]{4-4\sqrt{b}+b}$
$\iff \dfrac{4}{a}+\sqrt[3]{(2-\sqrt{b})(2+\sqrt{2}}=\sqrt[3]{(2+\sqrt{b})^2}+\sqrt[3]{(2-\sqrt{b})^2}$
Đặt
$\sqrt[3]{2+\sqrt{b}}=x$
$\sqrt[3]{2-\sqrt{b}}=y$
Ta có hệ sau:
$\left\{\begin{matrix} x^3+y^3=4\\ \frac{4}{a}+xy=x^2+y^2 \end{matrix}\right.$

 




#438150 Chứng minh rằng: $(x-1)(y-1)(z-1)\leq \frac{3}{...

Đã gửi bởi Simpson Joe Donald on 25-07-2013 - 19:50 trong Bất đẳng thức và cực trị

chỗ đó sao thế mình không hiểu

Thì ta có $(ab+bc+ca)^2\ge 3abc(a+b+c)$




#499343 $\large \left | x-3 \right |^{2013}+\left | x-2...

Đã gửi bởi Simpson Joe Donald on 16-05-2014 - 11:43 trong Phương trình, hệ phương trình và bất phương trình

Xét $x \geq 1$ => $VT>VP$ 
Xét $2< x<3$ => $VT<VP$ 
$x=3$ thì thoả mãn , $x=2$ cũng thoả mãn 
$x>3$ => vô nghiệm 
Vậy $x \in {2;3}$




#443778 Tìm Max: A=\frac{x}{(x+10)^{2}}

Đã gửi bởi Simpson Joe Donald on 18-08-2013 - 06:52 trong Bất đẳng thức và cực trị

Tìm max $A= \frac{x}{\left ( x+10 \right )^{2}}$

$A=\dfrac{x}{x^2+20x+100}=\dfrac{1}{x+20+\dfrac{100}{x}}\le \dfrac{1}{40}$